Difference between revisions of "1982 USAMO Problems/Problem 3"
(Created page with "== Problem == If a point <math>A_1</math> is in the interior of an equilateral triangle <math>ABC</math> and point <math>A_2</math> is in the interior of <math>\triangle{A_1BC}</...") |
m (→Solution) |
||
(2 intermediate revisions by 2 users not shown) | |||
Line 9: | Line 9: | ||
== Solution == | == Solution == | ||
− | {{ | + | First, an arbitrary triangle <math>ABC</math> has isoperimetric quotient (using the notation <math>[ABC]</math> for area and <math>s = \frac{a + b + c}{2}</math>): |
+ | <cmath>\frac{[ABC]}{4s^2} = \frac{[ABC]^3}{4s^2 [ABC]^2} = \frac{r^3 s^3}{4s^2 \cdot s(s-a)(s-b)(s-c)} = \frac{r^3}{4(s-a)(s-b)(s-c)}</cmath> | ||
+ | <cmath>= \frac{1}{4} \cdot \frac{r}{s-a} \cdot \frac{r}{s-b} \cdot \frac{r}{s-c} = \frac{1}{4} \tan A/2 \tan B/2 \tan C/2.</cmath> | ||
+ | |||
+ | Lemma. <math>\tan x \tan (A - x)</math> is increasing on <math>0 < x < \frac{A}{2}</math>, where <math>0 < A < 90^\circ</math>. | ||
+ | |||
+ | Proof. <cmath>\tan x \tan (A - x) = \tan x \cdot \frac{\tan A - \tan x}{1 + \tan A \tan x} = 1 - \frac{1}{\cos^2 x (1 + \tan A \tan x)}</cmath> | ||
+ | <cmath>= 1 - \frac{2}{1 + \cos 2x + \tan A \sin 2x} = 1 - \frac{2}{1 + \sec A \cos (A - 2x)}</cmath> is increasing on the desired interval, because <math>\cos (A - 2x)</math> is increasing on <math>0 < x < \frac{A}{2}.</math> | ||
+ | |||
+ | Let <math>x_1, y_1, z_1</math> and <math>x_2, y_2, z_2</math> be half of the angles of triangles <math>A_1 BC</math> and <math>A_2 BC</math> in that order, respectively. Then it is immediate that <math>30^\circ > y_1 > y_2</math>, <math>30^\circ > z_1 > z_2</math>, and <math>x_1 + y_1 + z_1 = x_2 + y_2 + z_2 = 90^\circ</math>. Hence, by Lemma it follows that | ||
+ | <cmath>\tan x_1 \tan y_1 \tan z_1 = \tan (90^\circ - y_1 - z_1) \tan y_1 \tan z_1 > \tan (90^\circ - y_1 - z_2) \tan y_1 \tan z_2</cmath> | ||
+ | <cmath> > \tan (90^\circ - y_2 - z_2) \tan y_2 \tan z_2 = \tan x_2 \tan y_2 \tan z_2.</cmath> | ||
+ | Multiplying this inequality by <math>\frac{1}{4}</math> gives that <math>I.Q[A_1 BC] > I.Q[A_2 BC]</math>, as desired. | ||
== See Also == | == See Also == | ||
{{USAMO box|year=1982|num-b=2|num-a=4}} | {{USAMO box|year=1982|num-b=2|num-a=4}} | ||
+ | {{MAA Notice}} | ||
[[Category:Olympiad Geometry Problems]] | [[Category:Olympiad Geometry Problems]] |
Latest revision as of 22:29, 18 May 2015
Problem
If a point is in the interior of an equilateral triangle and point is in the interior of , prove that
,
where the isoperimetric quotient of a figure is defined by
Solution
First, an arbitrary triangle has isoperimetric quotient (using the notation for area and ):
Lemma. is increasing on , where .
Proof. is increasing on the desired interval, because is increasing on
Let and be half of the angles of triangles and in that order, respectively. Then it is immediate that , , and . Hence, by Lemma it follows that Multiplying this inequality by gives that , as desired.
See Also
1982 USAMO (Problems • Resources) | ||
Preceded by Problem 2 |
Followed by Problem 4 | |
1 • 2 • 3 • 4 • 5 | ||
All USAMO Problems and Solutions |
The problems on this page are copyrighted by the Mathematical Association of America's American Mathematics Competitions.